LEGAL MED 2

Published on December 2016 | Categories: Documents | Downloads: 239 | Comments: 0 | Views: 1260
of 35
Download PDF   Embed   Report

Comments

Content

1 / 35
Legal Medicine
1. The body vested by law to have general supervision and regulation over the practice of medicine is:
A. Commission on Higher Education
B. Department of Education
C. Professional Regulatory Commission
D. Board of Medical Education
Answer: C
Reference:
Chapter II - Licensure and Regulatory Laws; p.8
Medical Jurisprudence by Pedro Solis (1988 ed)
MPL:
0.5
2. Pedro Doctor graduated from the Pulangue College of Medicine and immediately used the title MD after his name. He is:
A. Committing malpractice
B. Academically entitled to use MD
C. Illegally practicing medicine
D. Falsely using MD after his name
Answer: B
Reference:
Chapter III – The Practice of Medicine; p.48
Medical Jurisprudence by Pedro Solis (1988 ed)
MPL:
0.5
3. Any person who performs any of the acts constituting practice of medicine but is not qualified to do so shall be:
A. Committing malpractice
B. Committing fraud
C. Illegally practicing medicine
D. Acting under extraordinary authority
Answer: C
Reference:
Chapter III – The Practice of Medicine; pp.38, 48
Medical Jurisprudence by Pedro Solis (1988 ed)
MPL:
0.5
4. Josefa’s daughter was suffering from diarrhea. She consulted Petra who told her to give Pedilayte - Mild 30, a household remedy as classified by
BFAD. Which describe appropriately Petra’s action?
:
A. Not considered an act constituting practice of medicine
B. Illegal practice of medicine
C. Malpractice acts
D. Acting under false pretenses
Answer: A
Reference:
Chapter III – The Practice of Medicine; p.41
Medical Jurisprudence by Pedro Solis (1988 ed)
MPL:
0.5
5. This prevents an individual from being admitted to the practice of medicine:
A. Age 19 years
B. Passed the board examination
C. Holder of a valid Certificate of Registration issued by the Board of Medical Education
D. Must not have been convicted of any offense involving moral turpitude
Answer: A
Reference:
Chapter II - Licensure and Regulatory Laws; p.19
Medical Jurisprudence by Pedro Solis (1988 ed)
MPL:
0.25
6. This act does not constitute practice of medicine:
A. Physical examination of the patient
B. Collection of professional fees
C. Diagnosis and treatment
D. Prescribing drugs
Answer: B
Reference:
Chapter III – The Practice of Medicine; p.39
Medical Jurisprudence by Pedro Solis (1988 ed)
MPL:
0.5
7. A patient with hepatoma was erroneously diagnosed as tuberculoma by a licensed physician. This was the result of the non-performance of the
necessary work-ups. The physician is guilty of:
A. Illegal practice of medicine
B. Dereliction of duty
C. Criminal negligence
D. Malpractice
Answer: D
Reference:
Chapter XI –Specific Acts of malpractice; p.263
Medical Jurisprudence by Pedro Solis (1988 ed)
MPL:
0.25

Legal Medicine and Medical Jurisprudence

2 / 35
8. Jose is the son a physician. Upon graduation and before passing the board exams, he put his name under a sign bearing his father’s name and
started seeing patients. A patient was mismanaged and suffered injury. Jose was sued. What is the fault committed?
A. Illegal practice of medicine against Jose
B. Malpractice vs. Jose
C. Malpractice vs. Jose’s father
D. No liability
Answer: A
Reference:
Chapter III – The Practice of Medicine; p.48
Medical Jurisprudence by Pedro Solis (1988 ed)
MPL:
0.25
9. Ernie Barko advised Petra, a patient with breast CA to strictly follow his special cancer diet. He advised against taking medications as the chemicals
therein conflicted with the principle of the diet. Petra died and Ernie was sued. Which of the following is correct:
A. Ernie illegally practiced medicine
B. Ernie should be held liable for the non-intake of medications by Petra
C. Ernie is not liable since the proximate cause of Petra’s death is cancer
D. Malpractice on the part of Ernie
Answer: A
Reference:
Chapter III – The Practice of Medicine; p.48
Medical Jurisprudence by Pedro Solis (1988 ed)
MPL:
0.25
10. Beautician Lucy pricks pimples and dishes out topical lotions to clients. Dr. Balat is losing out to Lucy so she accuses the latter of illegal practice of
medicine. Which of the following is correct?
A. Tigidig is performing a purely cosmetic function
B. Tigidig is illegally practicing medicine
C. Administrative charges should be filed with the PRC against Tigidig
D. Tigidig’s parlor should be padlocked by the police authorities
Answer: B
Reference:
Chapter III – The Practice of Medicine; p.48
Medical Jurisprudence by Pedro Solis (1988 ed)
MPL:
0.25
11. Precy Lore is a faith healer who beats up the sick to exorcise the evil spirits causing the disease. One follower who was injured sued her. Which of
the following statement is correct:
A. Freedom to act out one’s belief is a constitutionally protected right
B. Precy is guilty of inflicting physical injuries
C. Precy is practicing the tenets of her religion
D. Precy illegally practiced medicine
Answer: B
Reference:
Chapter III – The Practice of Medicine; pp.42-45
Medical Jurisprudence by Pedro Solis (1988 ed)
MPL:
0.25
12. Dr. Pedro is scheduled to preside over a CPC in the Medical School. He is late, so that on his own volition, countered the traffic flow in the opposite
lane to reach his destination. He was caught by a traffic officer. Which can he invoke?
A. Doctor’s right of way in heavy traffic
B. Right of way in response to an emergency
C. None
D. Right to violate minor traffic rules
Answer: C
Reference:
Chapter V – Rights of Physicians; p.92-93
Medical Jurisprudence by Pedro Solis (1988 ed)
MPL:
0.5
13. A person need not be a physician to hold this position:
A. Municipal Health Officer
B. City Health Officer
C. Department of Health Secretary
D. Medical Staff in a private hospital
Answer: C
Reference:
Chapter V – Rights of Physicians; p.94
Medical Jurisprudence by Pedro Solis (1988 ed)
MPL:
0.5
14. Jose, an 18 year-old married male, was diagnosed with ruptured acute appendicitis. Who should give the consent for surgery?
A. Jose
B. Consent by the wife is necessary
C. Jose’s parent
D. Grandparents in the absence of the parents
Answer: A
Reference:
Chapter VI – Rights of Patients; p.140-141
Medical Jurisprudence by Pedro Solis (1988 ed)
MPL:
0.5
Legal Medicine and Medical Jurisprudence

3 / 35
15. Dr. Kildare, an American surgeon is in the Philippines on holiday. He joined a group of Filipino surgeons in a medical mission performing operations
for the poor and the marginalized. Which of the following is correct?
A. He can invoke reciprocity as the basis for his performing surgery in the Philippines.
B. He can practice even without a certificate of registration from the Board of Medicine.
C. He is illegally practicing medicine in the country.
D. He needs to pass the local board exams before he can practice.
Answer: C
Reference:
Chapter II – Licensure and Regulatory Laws; p.22
Medical Jurisprudence by Pedro Solis (1988 ed)
MPL:
0.5
16. Dr. Rodrigo released records of patient Juana to Atty. Jose, Juana’s lawyer after the latter demanded the same in connection with a contemplated
suit. Which right of the patient may have been violated?
A. Privacy
B. Confidentiality
C. Right to peace of mind
D. No rights were violated since it involves litigation
Answer: B
Reference:
Chapter V – Rights of Patients; p.94
Medical Jurisprudence by Pedro Solis (1988 ed)
MPL:
0.5
17. You evaluated a 65-year old man with 3-month history of chest pains and fainting spells that you feel merit cardiac catheterization. After informing
him fully of the benefits, consequences, risks, etc. and he was able to demonstrate that he understands all of these, he refuses the intervention. What
should you do?
A. Respect his choice
B. Explore reasons for his decision and try to convince him again
C. Consider the discussion ended
D. Continue with the procedure nevertheless
Answer: A
Reference:
Chapter V – Rights of Patients; p.160
Medical Jurisprudence by Pedro Solis (1988 ed)
MPL:
0.5
18. A 60-year old patient has complaints of abdominal that is persistent but not extreme. Workup shows hepatoma. He just retired from a busy
professional career and he and his wife are about to leave for a round the world cruise. What should you do?
A. Exercise your therapeutic privilege and inform him later when he returns
B. Inform him of his condition now
C. Inform him when his condition worsens
D. Inform his wife first
Answer: B
Reference:
Chapter VI – Rights of Patients; p.153-157
Medical Jurisprudence by Pedro Solis (1988 ed)
MPL:
0.5
19. While driving in EDSA, Dr. Gerardo witnessed the shooting of a politician by a group of armed assailants. He was subpoenaed to appear in court
because of his as heavy practice. He relented nevertheless and appeared as a:
A. Ordinary witness
B. Expert witness
C. Hostile witness
D.
Reluctant witness
Answer: A
Reference:
Chapter XXI –Medical Witness and the Court; pp.440
Medical Jurisprudence by Pedro Solis (1988 ed)
MPL:
0.75
20. A 70-year old man suffered a myocardial infarction and is admitted to the hospital with a very poor prognosis. He asks you not to tell his wife as he
feels she will not be able to take it. What should you do?
A. Tell the wife since the exercise of therapeutic privilege applies to the patient only
B. Encourage the patient to open dialogue with his wife
C. Keep quiet
D. Ask the nurse to inform the wife
Answer: B
Reference:
Chapter VI – Rights of Patients; p.157-158
Medical Jurisprudence by Pedro Solis (1988 ed)
MPL:
0.5
21. Dr. Jane is connected with one of the RHUs of Abra. One of her task is to implement the family planning program of the government. She is a
devout Catholic and espouses only the rhythm method. Which of the following is correct?
A. She has the prerogative to choose treatment modalities to apply to patients.
B. She can invoke self-limitation to practice based on religious beliefs.
C. She must ask to be transferred to duties consistent with her beliefs.
D. She is liable for insubordination for not carrying out the duties of her position.
Answer: C
Reference:
Chapter V – Rights of Physicians; p.87
Legal Medicine and Medical Jurisprudence

4 / 35
MPL:

Medical Jurisprudence by Pedro Solis (1988 ed)
0.5

22. One of the following is not subject to legal execution if a physician loses a civil suit for malpractice.
A. Golf shares
B. Stocks and money in bank
C. Real estate property
D. Medical library, equipment and instruments
Answer: D
Reference:
Chapter V – Rights of Physicians; p.93
Medical Jurisprudence by Pedro Solis (1988 ed)
MPL:
0.5
23. This type of consent is not contemplated under Philippine law:
A. Oral
B. Written
C. Expressed Indirectly
D. Blanket
Answer: D
Reference:
1.Chapter VI – Rights of Patients; p138
Medical Jurisprudence by Pedro Solis (1988 ed)
2. RA 5504-Aids prevention and Control act of 1998
MPL:
0.5
24. A 40 wk. AOG pregnant woman gave birth to a dead baby girl with double cord coil, tight. The obstetrician failed to do a biological physical profile of
the fetus. She is guilty of:
A. Homicide
B. Infanticide
C. Medical malpractice
D. Criminal negligence
Answer: C
Reference:
Chapter XI –Specific Acts of malpractice; p.262
Medical Jurisprudence by Pedro Solis (1988 ed)
MPL:
0.25
25. Jose gave consent to surgically remove one of his kidneys which he intends to sell to a patient in Japan with chronic renal disease. Which one of the
following requisites was violated as to make the consent invalid?
A. Informed and enlightened
B. Obtained voluntarily
C. Subject matter must be legal
D. Enlightened
Answer: C
Reference:
Chapter VI –Rights of Patients; p.125-126
Medical Jurisprudence by Pedro Solis (1988 ed)
MPL:
0.25
26. Baby Jane was bleeding massively due to marked thrombocytopenia secondary to dengue. Blood transfusion was required but the parents refused
on religious grounds. What should the physician do?
A. Secure a court appointed guardian to get consent for treatment
B. Transfuse immediately as this is an emergency
C. Respect the parents’ refusal of the lifesaving treatment
D. Do nothing
Answer: B
Reference:
Chapter VI –Rights of Patients; p.147
Medical Jurisprudence by Pedro Solis (1988 ed)
MPL:
0.25
27. Dr. Tan refused to attend on several occasions a court hearing citing as reason that he has a lot of OPD patients to see. He may be cited for:
A. Direct contempt of court
B. Indirect contempt of court
C. Disrespect for the court
D. Unduly prioritizing his clinic practice
Answer: B
Reference:
Chapter XXI –Medical Witness and the Court; pp.459
Medical Jurisprudence by Pedro Solis (1988 ed)
MPL:
0.75
28. A resident physician followed the order of a consultant that at face value was grossly wrong and resulted in injury to the patient. Which is correct?
A. The resident clerk is not liable since he is merely a trainee under the supervision of the consultant
B. Both the consultant and resident are liable for poor patient care
C. The consultant is not liable since the discretion lies with the resident to either follow or not follow his advice
D. The resident is liable since he is the one who exercised the clinical judgment that injured the patient
Answer: B
Reference:
Chapter XI –Physician-Patient Relationship; pp.73-74
Medical Jurisprudence by Pedro Solis (1988 ed)
Legal Medicine and Medical Jurisprudence

5 / 35
MPL:

0.25

29. Jose, who is not a physician, affixed MD after his name to enable him to secure a loan with the bank. Under the Medical Act of he is guilty of:
A. Committing malpractice
B. Committing fraud
C. Illegally practicing medicine
D. Considered not to be practicing medicine
Answer: C
Reference:
Chapter III –The Practice of Medicine; pp.38, 48
Medical Jurisprudence by Pedro Solis (1988 ed)
MPL:
0.5
30. Dr. Jose is practicing in Manila. A subpoena was issued by the court for him to appear in a drug case in Tuguegarao. He failed to appear. Which of
the following is correct?
A. He need not comply with the subpoena.
B. He will be arrested to compel his appearance.
C. He will be cited for direct contempt.
D. He should be given compensation to enable him to appear.
Answer: A
Reference:
Chapter XXI –Medical Witness and the Court; pp.461
Medical Jurisprudence by Pedro Solis (1988 ed)
MPL:
0.25
31. Jose was stabbed and brought to Dr. Juan for management. Dr. Juan refused to treat Jose since he was a dermatologist. Which one of the following
is applicable?
A. Dr. Juan can invoke his right to choose his patients.
B. Dr. Juan must manage the stabbing victim.
C. It is illegal to refuse rendering medical service.
D. Dr. Juan is liable for damages.
Answer: B
Reference:
Chapter XI –Rights of Physicians; p.84
Medical Jurisprudence by Pedro Solis (1988 ed)
MPL:
0.5
32. A 27-year old woman who has previously received no prenatal care presents at term. On ultrasound she has a placenta previa, but she refuses
cesarean section for any reason. What should the physician do?
A. Operate since this is an emergency
B. Respect the patient’s wishes
C. Refer the matter to the ethics committee as this involves the life of the mother and the fetus
D. The physician should get a court order to force a cesarean section
Answer: C
Reference:
Code of Ethics of the Medical Profession; pp. 475-476
Medical Jurisprudence by Pedro Solis (1988 ed)
MPL:
0.5
33. In a malpractice case, Dr. Procopio was ordered by the court to produce at trial the patient’s charts, laboratory and x-ray results and other
documents under his ppossession and control. The process served on him to do this is called:
A. Ordinary subpoena
B. Subpena duces tecum
C. Subpoena ad testificandum
D. Court summons
Answer: B
Reference:
Chapter XXI –Medical Witness and the Court; pp.456-457
Medical Jurisprudence by Pedro Solis (1988 ed)
MPL:
0.75
34. Dr. Balat adopted a self-imposed limitation to practice dermatology only. She may not do one of the following:
A. Practice only dermatology
B. Limit practice to specific days and time
C. Refuse to treat an emergency
D. Refuse to treat a common cold
Answer: C
Reference:
Chapter XVI –Emergencies in Medical Practice; pp.379-380
Medical Jurisprudence by Pedro Solis (1988 ed)
MPL:
0.5
35. Dr. Jose operated on a patient and inadvertently left behind a clamp. Which malpractice doctrine is applicable in this case?
A. Doctrine of Forseeability
B. Fellow Servant
C. Res-Ipsa Loquitur
D. Captain of the ship
Answer: C
Reference:
Chapter XVI –Emergencies in Medical Practice; pp.239-240
Medical Jurisprudence by Pedro Solis (1988 ed)
MPL:
0.5
Legal Medicine and Medical Jurisprudence

6 / 35
36. A personal disqualification that may cause the revocation of a physician’s certificate of registration is:
A. Unprofessional conduct
B. Issuing false medical certificates
C. Immoral or dishonorable conduct
D. Guilty of an offense involving moral turpitude
Answer: C
Reference:
Chapter XVI –Liabilities of Physicians; pp.179
Medical Jurisprudence by Pedro Solis (1988 ed)
MPL:
0.25
37. Josefa had herself aborted by a midwife. She subsequently went to Dr. Juana who did a D&C to complete the abortion. Dr. Juana did not report the
matter to the police authorities. What is Dr. Juana’s liability if any?
A. No liability
B. Liable as principal for doing the D&C
C. Only administratively liable
D. Liable as accessory for not reporting
Answer: D
Reference:
Chapter VIII –Criminal Liabilities of Physicians; p.201
Medical Jurisprudence by Pedro Solis (1988 ed)
MPL:
0.25
38. Jose’s inability to pay Dr. Kildare his fees for services rendered may result in:
A. Automatic termination of the patient-MD relationship
B. Action for collection filed by the physician against the patient
C. Lessened physician liability for negligence
D. Abandonment
Answer: B
Reference:
Chapter V –Rights of Physicians; p.105
Medical Jurisprudence by Pedro Solis (1988 ed)
MPL:
0.5
39. In a case of botched up surgery, Dr. Pedor, the chairman of surgery at Marconi Medical Center was called in to testify on the procedure. He is giving
testimony as:
A. Percipient witness
B. Expert witness
C. Hostile witness
D. Defense witness
Answer: B
Reference:
Chapter XXI –Medical Witness and the Court; pp.454-455
Medical Jurisprudence by Pedro Solis (1988 ed)
MPL:
0.75
40. One of the following is not a liability resulting from the wrongful act by a physician:
A. Criminal
B. Moral
C. Civil
D. Administrative
Answer: B
Reference:
Chapter VII –Liabilities of Physicians; pp.171-172
Medical Jurisprudence by Pedro Solis (1988 ed)
MPL:
0.75
41. Dr. Pedro had surgical training in a government hospital but did not take the Philippine College of Surgeons specialty examination. Dr. Juan
completed his surgical training at UCLA Medical Center and upon his return was certified as Fellow of the Philippine College of Surgeons. Out of envy at
the high patient traffic of Pedro, Juan filed a complaint against Pedro with the PRC for false claims of the latter as a surgeon. One of the following is
correct:
A. Pedro must possess the degree of skill and knowledge possessed by a surgeon since he claims to be one
B. Specialty society affiliation is the only basis for one to be a specialist
C. Specialty society certification is superior to the specialist certification under the Department of Health System
D. Pedro is a only surgeon in practice
Answer: A
Reference:
Chapter IV –Physician-Patient Relationship; p.72
Medical Jurisprudence by Pedro Solis (1988 ed)
MPL:
0.5
42. An administrative complaint was filed with the PRC against a physician who was eventually found guilty. This may result in:
A. Reprimand, suspension or revocation of license to practice
B. Imprisonment for criminal negligence
C. Civil damages paid to the patient to compensate for injuries sustained
D. Need to be re-certified
Answer: A
Reference:
Chapter VIII –Liabilities of Physicians; p.178
Medical Jurisprudence by Pedro Solis (1988 ed)
MPL:
0.5
Legal Medicine and Medical Jurisprudence

7 / 35
43. Dr. P. Alikero’s wife filed an administrative complaint before the PRC against her husband who was living with a mistress for having abandoned her
and her three children. The doctor’s acts are grounds for revoking his registration certificate and classifed as:
A. Personal disqualification
B. Unprofessional conduct
C. Unethical conduct
D. Criminal act
Answer: A
Reference:
Chapter VIII –Liabilities of Physicians; p.179
Medical Jurisprudence by Pedro Solis (1988 ed)
MPL:
0.5
44. One of the following does not necessarily terminate the patient-physician relationship and will render the physician liable:
A. Recovery
B. Discharge against advice
C. Discharge by the physician
D. Abandonment
Answer: D
Reference:
Chapter IV –Physician-Patient Relationship; p.77
Medical Jurisprudence by Pedro Solis (1988 ed)
MPL:
0.5
45. A gynecologist performed a total abdominal hysterectomy on a patient with myoma uteri. Before closing the abdomen, he discovered that the patient
had cancer of the cecum so he referred the patient to a general surgeon to perform the resection. Three days post-op, the patient had to be
reopened and a piece of gauze was found. Who may be held liable under the Captain of the ship Doctrine?
A. The operating room nurse
B. The second surgeon
C. The gynecologist
D.
he anesthesiologist
Answer: C
Reference:
Chapter VIII –Doctrines Applied in Malpractice Cases; p.237
Medical Jurisprudence by Pedro Solis (1988 ed)
MPL:
0.5
46. A physician must appear in response to a subpoena if his place of residence is how many kilometers from the court issuing the subpoena?
A. 100 kilometers
B. 50 kilometers
C. less than 50 kilometers
D. must appear no matter what the distance
Answer: C
Reference:
Chapter XXI –Medical Witness and the Court; p.461
Medical Jurisprudence by Pedro Solis (1988 ed)
MPL:
0.25
47. The receptionist in the hospital lobby receives a fee from Dr. No for referring cases to the latter. This fee, which is unethical, is called:
A. Contigent fee
B. Dichotomous fee
C. Retainer fee
D. Referral fee
Answer: B
Reference:
Chapter VIII –Rights of Physicians; p.102
Medical Jurisprudence by Pedro Solis (1988 ed)
MPL:
0.5
48. Dr. Jose refused to render service in an emergency case in an NPA controlled territory. Which is correct?
A. His license will be revoked.
B. He will be civilly liable for the injuries sustained by the patient for his failure to treat.
C. He can refuse service if there is risk on his life.
D. He violated his professional oath.
Answer: C
Reference:
Chapter XVI –Emergencies in Medical Practice; p.388
Medical Jurisprudence by Pedro Solis (1988 ed)
MPL:
0.5
49. While being treated for gunshot injuries, Abu Sabayad confided to Dr. Sharon that he was responsible for the kidnapping and beheading of twenty
hostages. Sabyad was captured and Dr. Sharon was called to testify. What should he do?
A. Refuse to testify to protect his life
B. He must not testify as the information is privileged
C. He must testify in the interest of justice, public safety and welfare as this involves a criminal matter
D. Feign loss of memory about the incident
Answer: C
Reference:
Chapter XXI –Medical Witness and the Court; p.446
Medical Jurisprudence by Pedro Solis (1988 ed)
MPL:
0.75
Legal Medicine and Medical Jurisprudence

8 / 35
50. What is legal is not necessarily ethical. The contraceptive methods legally allowed in the Philippines are the following except:
A. Use of physical barrier
B. Coitus interruptus
C. Elective Abortion under the pro-choice paradigm
D. Surgical sterilization
Answer: C
Reference:
Chapter XIX –Fertility Control pp.414-415
Medical Jurisprudence by Pedro Solis (1988 ed)
MPL:
0.5
51. As a result of the deformity he sustained from a facelift operation, Juana sued Dr. Hannibal for mental anguish, social humiliation and moral shock.
The type of damages she can recover as a result of these is:
A. Actual damages
B. Compensatory damages
C. Exemplary damages
D. Moral damages
Answer: D
Reference:
Chapter XV–Damages; p.372
Medical Jurisprudence by Pedro Solis (1988 ed)
MPL:
0.5
52. Which one of the following points should be considered in determining defloration:
A. Labias are in close contact with one another
B. Normal V shape configuration of the fourchette has been lost
C. Sharpness of the rugosities of the vaginal canal
D. Hymen is intact
Answer: B
Reference:
Chapter XXI–Virginity and Defloration; p.490-495
Legal Medicine by Pedro Solis (1987 ed)
MPL:
0.5
53. The following are true of virginity:
A. Refers to females
B. Applicable to males and females
C. Genital organs altered by carnal knowledge
D. A woman of virtue
Answer: A
Reference:
Chapter XXI–Virginity and Defloration; p.485
Legal Medicine by Pedro Solis (1987 ed)
MPL:
0.25
54. The presence of an intact hymen with distinct and regular edges and where the opening barely admits tip of the smallest finger even if the thighs are
separated is seen in:
A. Virgo intacta
B. True physical virginity
C. Semi-virginity
D. False physical virginity
Answer: B
Reference:
Chapter XXI–Virginity and Defloration; p.486
Legal Medicine by Pedro Solis (1987 ed)
MPL:
0.5
55. Carnal knowledge means:
A. Knowledge about the nature of sexual life
B. Rupture of the hymen
C. Slightest penetration of the female sexual organ by the male sexual organ
D. Full penetration is necessary
Answer: C
Reference:
Chapter XXI–Virginity and Defloration; p.500-501
Legal Medicine by Pedro Solis (1987 ed)
MPL:
0.25
56. One of the following is not a complication of hymenal lacerations:
A. Secondary infection
B. Hemorrhage
C. Fistulae formation
D. Impotence
Answer: D
Reference:
Chapter XXI–Virginity and Defloration; p.494-495
Legal Medicine by Pedro Solis (1987 ed)
MPL:
0.5
57. Josefa is 10 years old. She gave consent to have carnal knowledge with a male tourist provided she is paid money. Which is true?
A. Josefa is a victim of statutory rape.
B. Josefa is a child prostitute and thus no rape occurred.
Legal Medicine and Medical Jurisprudence

9 / 35
C.
D.
Answer: A
Reference:
MPL:

There is no rape since consent was given by Josefa.
Josefa must be 12 years old to be a victim of statutory rape
Chapter XXI–Sex Crimes; p.503
Legal Medicine by Pedro Solis (1987 ed)
0.5

58. This type of injury need not be reported to police authorities under PD 169:
A. Serious physical injuries
B. Mutilation
C. Slight physical injuries
D. Less serious physical injuries
Answer: C
Reference:
Chapter IX–Medico Legal Aspects of Physical Injuries; p.252
Legal Medicine by Pedro Solis (1987 ed)
MPL:
0.25
59. Petra scratched Juana in the face in an altercation. The doctor attended to the injury that resolved in less than 10 days but a resultant keloid scar
formation. The legal classification of this injury is:
A. Mutilation
B. Slight injuries
C. Serious physical injuries
D. Less serious injury
Answer: C
Reference:
Chapter IX–Medico Legal Aspects of Physical Injuries; p.248
Legal Medicine by Pedro Solis (1987 ed)
MPL:
0.5
60. Dr. Pedro, the MHO of Aurora Municipio shall perform an autopsy on a dead body when:
A. Requested by any relative of the victim
B. Verbally directed by the police
C. Upon order of the mayor
D. Ordered by the Provincial Hospital Medical Director
Answer: C
Reference:
Chapter V–Medico Legal Investigation of Death; p.165
Legal Medicine by Pedro Solis (1987 ed)
MPL:
0.25
61. Pedophilia is a form of sexual perversion where a man has a compulsive desire to seek gratification by:
A. Gazing at himself in the mirror
B. Having sexual intercourse with a child of either sex
C. Sexual intercourse with a corpse
D. Engaging in sexual intercourse with an animal
Answer: B
Reference:
Chapter XX1–Sexual Abnormalities; p.531
Legal Medicine by Pedro Solis (1987 ed)
MPL:
0.75
62. Dr. Acula is examining blood found in the crime scene. What is the source when blood is dark red in color and has a low oxygen content:
A. Child
B. Menstrual
C. Venous
D. Arterial
Answer: C
Reference:
Chapter III–Medico Legal Aspects of Identifcation; p.104
Legal Medicine by Pedro Solis (1987 ed)
MPL:
0.5
63. Bumper fracture of the leg bones in pedestrian-vehicle collision is a consequence of:
A. Secondary impact
B. Hit and run injuries
C. Primary impact
D. Run over injuries
Answer: C
Reference:
Chapter XVIII–Automotive Crash or Accident; p.464
Legal Medicine by Pedro Solis (1987 ed)
MPL:
0.5
64. A wound which is a result of a person’s instinctive reaction of self protection is what special type of wound:
A. Defense
B. Self-inflicted
C. Offense
D. Patterned
Answer: A
Reference:
Chapter IX–Medico Legal Aspects of Physical Injuries; p.244-245
Legal Medicine and Medical Jurisprudence

10 / 35
MPL:

Legal Medicine by Pedro Solis (1987 ed)
0.5

65. The condition where the body becomes soft and flaccid after the disappearance of rigor mortis is called:
A. Stage of secondary flaccidity
B. Decomposition
C. Suggillation
D. Putrefaction
Answer: A.C
Reference:
Chapter IV-Medico Legal Aspects of Death; p.130
Legal Medicine by Pedro Solis (1987 ed)
MPL:
0.5
66. Which of the following type of abortion, whether performed by a health professional or not, is criminal and punishable by law?
A. Intentional
B. Spontaneous
C. Consequence of a therapeutic act
D. Threatened
Answer: A
Reference:
Chapter IX–Abortion; p.558
Legal Medicine by Pedro Solis (1987 ed)
MPL:
0.5
67. The court ordered Dr. Juan to produce the skull of the victim who was killed by a blow on the head caused by a blunt weapon. The skull is what type
of evidence?
A. Autoptic evidence
B. Testimonial evidence
C. Documentary evidence
D. Experimental evidence
Answer: A
Reference:
Chapter I–Medical Evidence; p.12
Legal Medicine by Pedro Solis (1987 ed)
MPL:
0.25
68. If the person died of communicable disease, the body must be buried within this period of time unless the local health authorities permits otherwise:
A. 24 hours
B. 18 hours
C. 12 hours
D. 6 hours
Answer: A
Reference:
Chapter VIII-Disposal of the Dead Body; p.221
Legal Medicine by Pedro Solis (1987 ed)
MPL:
0.5
69. The race of a living person may be presumed in the features of the face. A person with thick lips and prominent eyes is presumed a:
A. Mongolian
B. Caucasian
C. Malay
D. Negro
Answer: D
Reference:
Chapter III–Medico Legal Aspects of Identification; p.48
Legal Medicine by Pedro Solis (1987 ed)
MPL:
0.5
70. Jose was hit by a truck in the legs and was thrown 10 feet from impact and hit the graveled part of the road. Dr. Jose noted abrasions, contusions,
lacerations in other parts of the body aside form the leg fractures. What type of injuries are those?
A. Hit and run injuries
B. Secondary impact injuries
C. Run over injuries
D. Primary injuries
Answer: B
Reference:
Chapter XVIII–Automotive Crash or Accident; p.465
Legal Medicine by Pedro Solis (1987 ed)
MPL:
0.5
71. Which of the following characteristics of a person is not easily changed:
A. Hair pattern
B. Places frequented
C. Speech, gait and mannerisms
D. Grade or profession
Answer: C
Reference:
Chapter III–Medico Legal Aspects of Identification; pp.43-44
Legal Medicine by Pedro Solis (1987 ed)
MPL:
0.5
Legal Medicine and Medical Jurisprudence

11 / 35
72. Part of the body that putrefies late:
A. Brain
B. Heart
C. Spleen
D. Liver
Answer: B
Reference:
Chapter IV-Medico legal Aspects of Death; p.140
Legal Medicine by Pedro Solis (1987 ed)
MPL:
0.5
73. Blood that does not clot, acidic in pH, with the presence of epithelial cells and Doderlain microorganisms is probably:
A. Blood from a hemophiliac
B. Menstrual blood
C. Blood from a person with metabolic acidosis
D. Septic blood
Answer: B
Reference:
Chapter III–Medico Legal Aspects of Identification; p.104
Legal Medicine by Pedro Solis (1987 ed)
MPL:
0.5
74. Which is true regarding exit wounds?
A. Always present
B. Bend to be smaller than the corresponding entrance wounds
C. Margins are usually everted, unbruised and unabraded
D. Presence powder tattooing
Answer: C
Reference:
Chapter XIII–Gunshot Wounds; p.361
Legal Medicine by Pedro Solis (1987 ed)
MPL:
0.25
75. Postmortem lividity may indicate the cause of death based on the color of the stain EXCEPT:
A. Bluish violet in asphyxia
B. Cherry red in CO poisoning
C. Chocolate in dichromate poisoning
D. Yellow in myocardial infarction
Answer: D
Reference:
Chapter IV–Medico Legal Aspects of Death; p.133
Legal Medicine by Pedro Solis (1987 ed)
MPL:
0.5
76. Republic Act 8353 has amended certain provisions of the old law on rape. These change include the following EXCEPT:
A. Rape is a crime against persons
B. Degenderization of the crime of rape
C. Allows for the concept of marital rape
D. Rape is a crime against chastity
Answer: C
Reference:
RA 8353-Law on Rape
MPL:
0.25
77. The body should be buried within 12 hours if this condition is present:
A. Body was not embalmed
B. Subject of legal investigation
C. Death due to dangerous communicable disease
D. Death due to violent crimes
Answer: C
Reference:
Chapter VIII-Disposal of Dead Body; p.220
Legal Medicine by Pedro Solis (1987 ed)
MPL:
0.5
78. One of the following is no longer true of rape under RA 8353:
A. Offender may be any person
B. Offended party must be female
C. Insertion of the penis into another person’s mouth or anus
D. Insertion of any instrument or object into the genital and anal orifice of another person
Answer: B
Reference:
RA 8353-Law on Rape
MPL:
0.25
79. Josefa does not intend to hurt her child, Jose. When she is drunk, she periodically batters Jose. She becomes remorseful afterwards and provides
proper care in between her drinking spree. What type of child abuser is Josefa?
A. Occasional child abuser
B. Constant child abuser
C. Intermittent child abuser
D. One time child abuser
Answer: A
Legal Medicine and Medical Jurisprudence

12 / 35
Reference:
MPL:

Chapter XX–Child Abuse; p.481
Legal Medicine by Pedro Solis (1987 ed)
0.25

80. This finding in death is not consistent and may or may not appear on a person who died:
A. Rigor Mortis
B. Algor Mortis
C. Putrefaction
D. Cadaveric Spasm
Answer: D
Reference:
Chapter IV-Medico legal Aspects of Death; p.129
Legal Medicine by Pedro Solis (1987 ed)
MPL:
0.5
81. Pedro calls people up on the phone in the early hours of the morning and starts using obsecene and foul language to obtain sexual gratification. This
sexual deviation is known as:
A. Pedophila
B. Masochism
C. Coprolalia
D. Exhibitionism
Answer: C
Reference:
Chapter XXI–Sexual Abnormalities; p.536
Legal Medicine by Pedro Solis (1987 ed)
MPL:
0.5
82. The lead agency tasked to handle cases of child abuse is:
A. Department of Health
B. Department of Justice
C. Department of Social Welfare and Development
D. DILG
Answer: C
Reference:
Chapter XX–Child Abuse; p.482
Legal Medicine by Pedro Solis (1987 ed)
MPL:
0.25
83. One of the following is not helpful in determining the sex of the skeleton:
A. Skull
B. Sternum
C. Cranial sutures
D.
Pelvis
Answer: C
Reference:
Chapter III-Medico Legal Aspects of Identification; p.80
Legal Medicine by Pedro Solis (1987 ed)
MPL:
0.5
84. Dactylography is...
A. The study of the pores on the papillary or friction ridges of the skin for the purpose of identification
B. Art of changing fingerprints
C. Art and study of recording fingerprints as a means of identification
D. Art of comparing fingerprints for identification
Answer: C
Reference:
Chapter III-Medico Legal Aspects of Identification; p.56
Legal Medicine by Pedro Solis (1987 ed)
MPL:
0.5
85. An idiot is exempted from criminal liability. An idiot has an IQ of:
A. 0 – 20
B. 40 – 70
C. 20 – 40
D. 70 – 80
Answer: A
Reference:
Chapter XXXII-Disturbance of Mentality; p.643
Legal Medicine by Pedro Solis (1987 ed)
MPL:
0.5
86. This finding suggests suicide:
A. Indication of struggle
B. Presence of hesitation cuts or tentative incisions
C. Multiple wounds in different parts of the body
D. No history of depression
Answer: B
Reference:
Simpson’s Forensic Medicine, 11th edition; p.76
Legal Medicine by Pedro Solis (1987 ed)
MPL:
0.5
Legal Medicine and Medical Jurisprudence

13 / 35
87. A false interpretation of external stimulus which may be manifested with the sense of sight, hearing, taste touch and smell is:
A. Hallucination
B. Delusion
C. Illusion
D. Obsession
Answer: C
Reference:
Chapter XXXII-Disturbance of Mentality; p.629
Legal Medicine by Pedro Solis (1987 ed)
MPL:
0.5
88. Disorientation, mental confusion, dizziness, sensory disturbances exaggerated emotional state suggest that the blood alcohol level is:
A. 150-300mg%
B. 400 mg%
C. 100 mg%
D. 20 mg %
Answer: A
Reference:
Chapter XXXIII-Alcoholism; p.695
Legal Medicine by Pedro Solis (1987 ed)
MPL:
0.5
89. The medico-legal importance of postmortem lividity is:
A. Reliable sign of death
B. Determines exact time of death
C. Affects all portions of the body
D. Development of skin color suggest injury
Answer: A
Reference:
Chapter IV-Medico legal Aspects of Death; p.132
Legal Medicine by Pedro Solis (1987 ed)
MPL:
0.5
90. If this is present, organs for transplantation may not be harvested:
A. No respiration
B. No blood pressure
C. Heartbeat and pulse absent
D. Reversible coma
Answer: C
Reference:
Chapter IV-Medico legal Aspects of Death; p.113
Legal Medicine by Pedro Solis (1987 ed)
MPL:
0.5
91. Head injury which may be found in areas of the skull offering the least resistance is known as:
A. Locus minoris resitencis
B. Countre cuop injuries
C. Remote inuires
D. Coup injuries
Answer: A
Reference:
ChapterXI-Injuries in Different Parts of the Body; p.301
Legal Medicine by Pedro Solis (1987 ed)
MPL:
0.25
92. The chemical change of body fat which takes place in wet conditions and results in a grayish, greasy or brittle compound is known as:
A. Mummification
B. Adipocere formation
C. Bacterial Putrefaction
D. Lytic changes
Answer: B
Reference:
Simpson’s Forensic Medicne, 11th edition; p.29
MPL:
0.25
92.

Indentations left in soft or pliable materials belong to what type of fingerprints:
A. Latent
B. Plastic
C. Visible
D. Imprinted
Answer: B
Reference:
Chapter III-Medico Legal Aspects of Identification; p.59
MPL:
0.5
94. A DNA typing method where the small sample size is chemically amplified to produce large amounts is known as the....
A. Restriction Fragment Length Polymorphism (RFLP) Method
B. Phenolphthalein Test
C. Polymerase Chain Reaction (PCR) Method
D. Precipitate Ring Test Method
Answer: C
Reference:
Simpson’s Forensic Medicne, 11th edition; p.42
Legal Medicine and Medical Jurisprudence

14 / 35
MPL:

0.25

95. The absence of smoke soiling, burning or powder tattooing is suggestive of this type of gunshot wound:
A. contact wound
B. close range wound
C. Long range wound
D. mid range wound
Answer: C
Reference:
Simpson’s Forensic Medicne, 11th edition; p.69
MPL:
0.25
96. Injury causation in torture include the following:
A. sexual assaults
C. electrical shock
B. ligatures
D. all of the above
Answer: D
th
Reference:
Simpson’s Forensic Medicne, 11 edition; p.140-141
MPL:
0.75
97. For a child to be legitimate, he must be born in lawful wedlock or within these number of days after the dissolution of marriage:
A. 120 days
B. 180 days
C. 300 days
D. 360 days
Answer: C
Reference:
Chapter XXVII-Paternity and Filiation: p. 588
Legal Medicine by Pedro Solis (1987 ed)
MPL:
0.25
98. Pedro is impotent yet her wife gave birth to a healthy baby while in lawful wedlock. Which is true?
A. The child’s legitimacy cannot be questioned
B. Proof of impotence will overthrow presumption of legitimacy
C. The child upon birth can be declared illegitimate
D. All of the above
Answer: B
Reference:
Chapter XXX-Impotence and Sterility; p. 620
Legal Medicine by Pedro Solis (1987 ed)
MPL:
0.5
99. Jose was distraught at being found guilty of the heinous crime of raping his daughter and sentenced to death. He subsequently became insane.
Which should rake place?
A. He should be executed.
B. His execution should be stayed until he recovers.
C. He should undergo another trial upon recovery of sanity
D. He should be pardoned.
Answer: B
Reference:
Chapter XXXI-Disturbance of Mentality; p.627
Legal Medicine by Pedro Solis (1987 ed)
MPL:
0.5
100. The medico legal classification of death due to injuries inflicted upon the body by some form of outside force which is the proximate cause thereof
is:
A. Natural
B. Violent
C. Sickness caused
D. Instantaneous
Answer: B
Reference:
Chapter XXVII-Causes of Death; p. 185
Legal Medicine by Pedro Solis (1987 ed)
MPL:
0.25
101. Tardieu spots are associated with:
A. Hanging
C. ligature strangulation
B. Manual strangulation
D. choking
Answer: A
Reference:
Chapter XXVII-Death By Asphyxia; p. 433
Legal Medicine by Pedro Solis (1987 ed)

1.

The most common preparation of Amphetamine in the market are the following, EXCEPT:
A. pearly gates
C. peaches
B. co-pilot
D. foot balls

2.

A state of psychic or physical need to administer a substance periodically or continuously is:
A. drug addiction
C. drug dependence
Legal Medicine and Medical Jurisprudence

15 / 35
B.

drug habituation

D. all of the above are correct

3.

When the victim of a rape becomes insane after the act, the penalty is:
A. death
C. prison correctional
B. reclusion perpetua
D. reclusion temporal

4.

Based on the action and effect on the body, which of the following is a cerebral depressant?
A. Aconite
C. digitalis
B. Camphor
D. hydrogen cyanide

5.

A lawyer, who is unmarried, had an illicit relation with his unmarried secretary. He is guilty of:
A. Immorality
C. concubinage
B. Adultery
D. reckless imprudence

6.

Which of the following examinations is the most widely accepted but quite impractical way to determine the concentration of alcohol in the
body?
A. Saliva
C. breath
B. Urine
D. blood

7.

A sudden and irresistible force compels a person to indulge in intoxication which is NOT voluntary is called:
A. Multilomania
C. pyromania
B. Kleptomania
D. dypsomania

8.

The method where breath is used to analyze alcohol is:
A. enzymatic method
C. infrared solution method
B. gas chromatographic method
D. chemical method

9.

Kunkel’s test is carried out on a blood sample to determine the presence of:
A. hydrogen sulfide
C. carbon monoxide
B. hydrogen cyanide
D. nitrates

10. Death due to intake of adulterated liquor is attributed to:
A. glacial acetic acid
C. isopropyl alcohol
B. ethylene glycol
D. ethyl alcohol
11. The basis to pronounce a person dead:
A. molecular death
C. somatic death
B. apparent death
D. cellular death
12. The corneal reflex to light stimulus may be absent in a living patient in the following conditions, EXCEPT:
A. Epilepsy
C. meningitis
B. Uremia
D. under general anesthesia
13. A wound may be indirectly fatal by reason of:
A. scarring effect
C. mechanical injuries to vital organs
B. hemorrhage
D. shock
14. Which of the following will produce a lacerated wound?
A. change of atmospheric pressure
B. heat

C. chemical explosion
D. tearing force

15. The presence of many stab wounds inflicted on a victim who died points to:
A. serious physical injuries
C. murder
B. homicide
D. parricide
16. The crime of mutilation is understood to mean:
A. inflicting physical injury
B. rendering any part of the body useless

C. intentionally cutting off any part of the body
D. accidentally depriving a person of any part of the body

17. A post-mortem wound differs from ante-mortem wound in that:
A. the wound edges gape
B. the tissues around the wound are deeply situated
C. there is a lot of blood present
D. there is no clotting of blood
18. The most important determinant of the wounding capacity of a bullet is:
A. bullet momentum
C. bullet caliber
B. bullet shape
D. presence or absence of a “jacket”
19. The physical phenomenon when there is more “shocking or knock down power” of the bullet is:
A. shock wave
C. muzzle blast in contact fire
B. disintegration of bullet
D. hydrostatic force
20. The production of the furrow in the cranium called gutter fracture is caused by:
A. tangential approach of the bullet
Legal Medicine and Medical Jurisprudence

16 / 35
B.
C.
D.

simple compression of the skull
impact of the blunt object
osteoporosis of the cranium

21. How soon after drowning does the dead body float?
A. within 24 hours
C. after 48 hours
B. after 36 hours
D. after 60 hours
22. The following factors influence the floating of a body in water, EXCEPT:
A. Obesity
C. color of the skin
B. Age
D. season of the year
23. A whitish foam in the mouth and nostrils as a sign of drowning is :
A. Emphysema
C. edema aquasum
B. tache noire
D. champignon d’ocume
24. Which of the following findings point to drowning as a cause of death?
A. blood chloride in the left ventricle is the same as in the right ventricle
B. washerwoman’s hand and feet
C.
physical injuries indicative of a struggle
D. marked congestion of the trachea and bronchi which are filled with foreign bodies
25. Cutis Galina or washerwoman’s hand and feet on a corpse signifies that:
A. the body has been in water for some time
B. the body is that of a laundry woman
C. the body died of drowning
D. the body died of syphilis
26. “Tete de negri” or the bronze coloration of the head and neck of a dead person is observed in:
A. garroting
B. suicide by drowning
C. downing
D. injury to the head caused by a blunt instrument
27. Accidental death of a young child by suffocation either from the pressure of the beddings and pillows or from the pressure of the unconscious
or drunk mother is called:
A. battering
B. overlaying
C. burking
D. mugging
28. Covering both nostrils and mouth by a pillow may cause death due to:
A. traumatic asphyxia
B. asphyxia by choking
C. asphyxia by smothering
D. asphyxia by mugging
29. A special form of strangulation with the assailant standing at the back of the victim and the forearm is applied in front of the neck is called:
A. throttling
B. palmar strangulation
C. mugging
D. gagging
30. A form of asphyxial death wherein pressure on the trachea is caused by placing the neck at the bend of the elbow is called:
A. overlaying
B. burking
C. garroting
D. mugging
31. The cause of death in wrestling where the knees of the offender is applied on the victim’s neck is:
A. overlaying
B. mugging
C. burking
D. garroting
32. Mugging belongs to what classification of asphyxia?
A. hanging
B. suffocation
C. strangulation
D. traumatic crush
33. The following are post-mortem findings in death from asphyxia, EXCEPT:
A. lungs are engorged with dark blood
B. congestion of the spinal cord
C. congestion of abdominal viscera
Legal Medicine and Medical Jurisprudence

17 / 35
D.

subepicardial petechial hemorrhages

34. The type of asphyxial death when oxygen is delivered to the tissues but cannot be utilized properly due to failure of cellular oxidative process
is:
A. Anoxic
C. anemic anoxic
B. Histotoxic
D. stagnant anoxic
35. Which of the following types of asphyxial death is classified as anoxic death?
A. traumatic crush asphyxia
B. shock
C. carbon monoxide poisoning
D. cyanide poisoning
36. One hundred eighty-one days (181) days after the wedding, the child is born to Maria by his former live-in partner. The child is considered:
A. natural child
B. illegitimate
C. legitimate
D. legitimated
37. A child born out of lawful wedlock where the parent eventually had a valid marriage is considered:
A. incestuous
B. sacrilegious
C. adulterous
D. legitimated
38. The children of a prostitute are called:
A. incestuous children
B. legitimate children
C. sacrilegious children
D. manceras children
39. A natural child (proper) may be legitimated. Which requisite is NOT correct?
A. there must be a subsequent marriage of the parents
B. the child must be acknowledged by the father
C. the child must be acknowledged by both parents before marriage
D. the child must be natural
40. “A “ was married to “B,” his own step-daughter, hence, the marriage is void. A child born thereafter is considered:
A. adulterous
B. natural by legal fiction
C. natural child by presumption
D. legitimated
41. The penalty for a young widow who remarries before the legal restriction s lifted:
A. reclusion perpetua
B. reclusion temporal
C. arresto menor
D. arresto mayor
42. Which is true about impotence?
A. A person who is impotent is sterile.
B. A sterile person is impotent.
C. Both sterility and impotence may exist at the same time.
D. All of the above are true
43. The demonstration of impotence is important:
A. as a ground for divorce
B. as a ground for legal separation
C. as evidence in defence of an alleged father in a paternity suit
D. as a disqualification from getting a marriage license
44. Marriage may be annulled based on the following conditions, EXCEPT:
A. the woman concealed a pregnancy of another man
B. consent of the man was by threat
C. elopement of parties
D. consent of the woman was taken by fraud
45. Artificial donor insemination with the consent of the husband but without the consent of the wife may result in:
A. charges of rape against the physician
B. action for divorce against the husband based on assault
C. a paternity suit against the donor
D. criminal assault against the physician
46. A mentally defective person with a mentality of a 2-7 years old and an IQ of 20-40 is a/an:
A. moron
Legal Medicine and Medical Jurisprudence

18 / 35
B.
C.
D.

zombie
imbecile
mongoloid

47. A genius or near-genius has an IQ of:
A. 100
B. 120-140
C. below 140
D. above 140
48. Insanity may be basis for the following, EXCEPT:
A. invalidation of a contract
B. annulment of marriage
C. voidance of a will
D. paternity of a pregnancy
49. A petition for the confinement of an insane person must be filed by:
A. director of health
B. mother of the person
C. chief of police
D. trial judge
50. The crime committed by the accused when she killed her husband while suffering from malignant malaria that caused acute melancholia is:
A. homicide
B. not criminally liable
C. parricide
D. murder
51. Curren’s rule is concerned with:
A. time of death
B. mental illness
C. malpractice liability
D. court procedure
52. An erroneous perception without an external object of stimulus is called:
A. hallucination
B. daydreaming
C. fantasy
D. illusion
53. What type of delusion occurs when one thinks that he is always the subject matter of conversation although it is not a fact?
A. reference
B. nihilistic delusion
C. persecution
D. self-accusation
54. Excessive and irrational fear of sacred things is called:
A. hierophobia
B. heliophobia
C. bibliophobia
D. aerophobia
55. Abnormal mental condition whereby a person is performing an act while in the state of natural sleep is:
A. semisomnolnece
B. somnolencia
C. delirium
D. somnambulism
56. Which of the following statement or principle is TRUE?
A. Right may always be waived.
B. Customs are always contrary to law.
C. Rules are repealed by subsequent ones.
D. All of the above
E. None of the above
57. Based on the manner of action and effect on the body, which poison belongs to the deliriant group of narcotics?
A. belladonna
B. synthetic hypnotics
C. opium
D. chloral
58. Under the Dangerous Drugs Act, which of the following is NOT classified as a dangerous drug?
A. codeine
B. LSD
C. Indian hemp
Legal Medicine and Medical Jurisprudence

19 / 35
D.

Paracetamol

59. Indian hemp is synonymous to:
A. LSD
B. Coca leaf
C. Marijuana
D. Amphetamine
60. Wood alcohol is otherwise known as:
A. methyl alcohol
B. ethylene glycol
C. isopropyl alcohol
D. ethyl alcohol
61. The site of remote action of arsenics is in the:
A. peripheral nerve
B. mucous membrane
C. liver
D. heart
62. The partial or complete disruption in the continuity of a muscular or ligamentous support of a joint is called:
A. subluxation
B. fracture
C. strain
D. sprain
63. A wounding agent producing communication between the inner and outer portion of the hollow organ results in:
A. deep wound
B. perforating wound
C. penetrating wound
D. superficial wound
64. When the wounding agent enters the body but did not come out, the wound is a:
A. perforating wound
B. superficial wound
C. deep wound
D. penetrating wound
65. Physical
usually:
A.
B.
C.
D.

injuries which develop on parts of the body when bone tissues are deep seated caused by forcible impact of hard blunt objects are
abrasion
contusion
hematoma
lacerated wound

66. When blood is exposed to the atmosphere, its hemoglobin is converted to hematin. Which statement is NOT correct?
A. blood of one week and that of six weeks may not present difference in physical and chemical properties
B. in warm weather it takes place within 24 hours
C. color is changed from red to bright scarlet
D. presence of acid accelerates formation of hematin
67. In starvation, the following factors influence the length of survival of a human body, EXCEPT:
A. children suffer earlier than old people
B. men can withstand starvation longer than woman
C. exposure to higher temperature will accelerate death
D. a healthy person can resist more deprival of food
68. In the identification of both the living and the dead before onset of decomposition, which statement is NOT correct?
A. birth marks like spots naevi may be remove by carbon dioxide snow, eletrocautery or excision
B. moles are permanent and cannot be removed except by surgery
C. certain occupations may result in some characteristic marks or identifying guide
D. blood type examination may be utilized to distinguish one person from another
69. The biologic test used to determine whether the blood is of human origin or not is known as:
A. hemin crystal
B. benzidine
C. precipitin
D. Takayama
70. A reddish brown, caked material was found on the sleeve of the suspect’s shirt. Any of the following tests may be used to determine whether the
stain is blood or not, EXCEPT:
A.
Schombein’s test
B.
Benzidine test
C.
Teichmann’s test
D.
Icard’s test
Legal Medicine and Medical Jurisprudence

20 / 35
71. Hair that is fine, deficient of pigments and devoid of medulla belongs to a/an:
A.
adolescent
B.
old person
C.
adult
D.
child
72. Fingerprint may persist if not removed at the scene for:
A.
days
B.
weeks
C.
months
D.
years

73. One way to determine whether hair is of human or animal origin is:
A. by determining the cephalic index
B. by the application of Schourup’s formula
C. by determination of medullary index
D. none of the above
74. Fingerprinting is considered to be the best valuable method of identification. It is universally used because fingerprints, until decomposition, are not
changeable from the time they are formed in the fetus during pregnancy in the:
A.
second month
B.
third month
C.
fourth week
D.
fifth month
75. One method to approximate the height of the cadaver with the extremities missing is to measure:
A. length of the head times eight
B.
the distance from the base of the skull to the coccyx which is 55% of the height
C.
the distance between the supra-sternal notch and the pubic symphysis which is one-half of the height
D.
length of the vertex of the skull to the pubic symphysis times three
76. A few points of differences between a male and a female pelvis are the following, EXCEPT:
A.
the male greater sciatic notch is narrow, the female wide
B.
the male obturator foramen is egg-shaped; the female triangular
C.
the male body of pubis is wider; the female narrow
D.
the male sacrum is short and narrow, the female long and wide
77. When a physician is required to determine the ages of the contracting parties for marriage, the analysis of the following are relevant, EXCEPT:
A.
presence of the 3rd molar past age of wisdom
B.
presence of pubic hair as evidence of maturity
C.
weight of parties
D.
development of the breasts as evidence of womanhood
78. In the ordinary method of human identification applicable to the living person, the “facies” is a characteristic not easily changed. Which appearance
of the face is indicative of approaching death?
A. Facies leonine
B.
Mongolian facies
C. Hippocratic facies
D.
Myxedemic facies
79. Scars have characteristics that may show the cause of the previous lesion. What is NOT valid?
A.
Flogging produces scars that are fine white lines diagonally across the back with depressed small spot at intervals
B.
A surgical operative scar has a regular form and situation with stitch marks
C.
Scars of burns and scalds are large, irregular in shape and may be keloid
D.
Gunshot wound leave scars that are irregular in shape, furrowed with edges hardened and uneven
80. In determining the sex of a skeleton, which bone is NOT used?
A.
pelvis
B.
skull
C.
sternum
D.
Tibia
81. Barr bodies observed in scraping of the mucus membrane of the mouth, a test to determine the sex of the body is:
A.
chromosomal test
B.
gonadal test
C.
genital test
D.
social test
82. A method of identification, wherein a verbal, accurate and picturesque description of the person identified are required is:
A. rougue’s gallery
B.
portrait parle
Legal Medicine and Medical Jurisprudence

21 / 35
C.
photographic file
D. Bertillon system
83. When can an examination be done to confirm the identity of a dead person in settling succession to an estate?
A. as soon as it can be legal
B.
after 10 years
C.
before the effects of embalming wears out
D.
after 5 years
84. Evidence obtained regarding the issue of the truth serum is not admissible in court because:
A. the information is obtained under duress
B.
the information given by the suspect is not voluntary since he is under the influence of a drug
C.
the truth serum is still in experimental stage
D.
the information is obtained through deceit
85. The declaration of a patient who knows that he is dying from cancer tells his physician that his compadre told him that he (compadre) killed his wife
is:
A.
hearsay evidence
B.
dying declaration
C.
testimonial evidence
D.
deposition
86. A statement made by a dying patient to the attending physician in reference to the persons who inflicted injury on him is known as:
A. privileged communication
B.
dying declaration
C.
hearsay evidence
D.
inadmissible evidence
87 The Keeler’s polygraph or lie detector to demonstrate evidence of emotional tension in a person who is lying records the following, EXCEPT:
A. psychogalvanic reflex
B. muscular tension
C.
respiration
D.
blood pressure
88. Standard medical textbooks are as a rule is not admissible as evidence in court because:
A. the author cannot be presented as a witness to be subjected to cross examination
B. the evidence presented is not the personal knowledge of the witness
C. it might prolong the proceeding by the presentation of the adverse party against the opinion of the author of the book
D. none of the above
89. It is NOT possible to determine the exact length of time a child has lived after birth. The following points may be given an approximate idea,
EXCEPT:
A. caput succedaneum in a child born with head presentation lasts up to the second day
B. the umbilical artery begins to contract at about 10 hours after birth
C. exfoliation of the skin in the abdomen occurs in the first 3 days after birth
D. body of a newborn child is bright red in color and covered with vernix caseosa up to 2 days
90. Physical evidence recovered from the crime scene such as a knife with fingerprints of the suspect is called:
A. circumstantial evidence
B. direct evidence
C. tracing evidence
D. corpus delict
91. A confession is different from admission because the latter is:
A. expressed acknowledgement of the truth of his guilt as to the crime charged
B. statement of fact which does not directly involve an acknowledgement of guilt
C. statement of guilt
D. all of the above
92. A woman took poison to commit suicide while her pregnancy is undetectable at 8 weeks. Timely medical intervention saves her life but ended in
abortion. Is she guilty of abortion or not?
A. She is not guilty of abortion or of suicide attempt if she is a psychiatric case
B. No, because she did not have the intention to abort
C. She is guilty of abortion but she is morally guilty of trying to commit suicide
D. Yes, because whatever she did, it contributed to abortion
93. Simulation of births and the substitution of one child for another performed by a physician is punishable by:
A.fine of not less than P1000
B. prison mayor and a fine of not less than P1000
C. Prison mayor
D. Prison mayor, a fine of not less than P1000 and temporary special disqualification
94. A physician who caused an abortion in a pregnant woman upon her request where the woman died, is liable for:
A.intentional abortion
B.intentional abortion with homicide
Legal Medicine and Medical Jurisprudence

22 / 35
C.homicide
D. murder
95. The following are signs of intra-uterine fetal death, EXCEPT:
A. HCG test positive
B. Absence of fetal movement
C. Ultrasound evidence of cranial bones overlapping
D. No progress in the enlargement of the uterus
96. Which of the following types of abortion is criminal and punishable by law?
A. intentional
B. post-mortem
C. spontaneous
D. therapeutic
97. The following are oxytocic substances that cause contractions of a pregnant uterus, EXCEPT:
A. quinine
B. progesterone
C. pitocin
D. ergotrate
98. A baby born alive has a legal personality. One of the tests used to determine life before death of a fetus is to float the lungs in water is:
A. Wrendin’s test
B. Icard’s test
C. Breslau’s test
D. Hydrostatic test
99. A physician who terminates a pregnancy of 8 weeks, is guilty of:
A. manslaughter
C. being an abortionist
B. parricide
D. Infanticide
100. A woman may develop menopause at 20 years old. The condition is called:
A. senility
C. menarche
B. premature aging
D. climacterium praecox

CEBU INSTITUTE OF MEDICINE
Legal Medicine, Jurisprudence and Bioethics

SUBJECT MATTER QUESTION
NO.
RECALL COMPRE
HENSION
APPLICA
TION
1. Legal Medicine 1 - 65
86 - 100 1, 3, 7 - 15, 17, 21 - 23, 25 - 28, 32 - 34, 36 - 40, 42 - 45, 48 - 51, 56, 57, 59 - 62, 65, 86 -100
46, 47, 52- 55, 58, 63
20, 64,

2, 4, 5, 6, 16, 18, 19, 24, 29 - 31, 35, 41,

2. Medical Jurisprudence
66 - 81

66, 69, 71 - 73, 77, 79, 80

67, 68, 70, 74, 76, 78, 81

75
3. Bioethics
82 - 85
82, 83
84, 85

MEAN MPL FOR THE EXAM = 67.35

Right/s of a suspect during custodial interrogation:
If he cannot afford an attorney, one will be appointed for him.
He has no right to consult an attorney.
Anything he says cannot be used against him.
All of the above
(Answer: A; page 34; MPL 90)
The choice of methods of questioning depend on:
having the suspect's right in mind.
understanding the suspect's rights.
psychological evaluation of the subject by the interrogator.
Legal Medicine and Medical Jurisprudence

23 / 35
all of the above
(Answer: C; page 34; MPL 60)
A public officer commits the crime of maltreatment of a prisoner by:
detaining him illegally.
imposition of punishment not authorized by regulation.
obtaining some information form the prisoner.
having a physician witness the interrogation.
(Answer: B; page 38; MPL 90)
A psychological offender is one who:
commits crimes for financial gains
is highly skilled
engages in a crime which requires special skills
commits crime with a motive
(Answer: A; page 34; MPL 60)
A non-emotional offender is one who:
commits crime in the heat of passion
has the sense of moral guilt
commits crime for financial gain
commits a crime without knowing the nature of the act.
(Answer: C; page 34; MPL 33)
The following qualifies homicide into murder:
killing of a wife or spouse
killing of a child who is three days' old
killing by means of fire
killing of a victim without justification
(Answer: C; page 195; MPL 60)
Unintentional abortion is caused by:
an act of the woman herself
violence inflicted on the woman
dispensing of abortive
the pregnant woman to conceal her pregnancy
(Answer: B; page 559; MPL 90)
Active aggressive offenders are known to be persons who commit a crime:
because of inducement
in an impulsive manner
which requires limited skill
in the heat of passion
(Answer: B; page 33; MPL 60)
Rational offenders are those who commit a crime with:
full possession of their faculties
with evidence of premeditation
motive
all of the above
(Answer: D; page 34; MPL 90)
An autopsy maybe performed:
on orders of a governor
upon written request of the attending physician
whenever required by special laws.
upon request of the barangay captain in behalf of the nearest of kin.
(Answer: C; page 165; MPL 90)

Medical evidence is admissible when:
the rules of evidence are the same in all courts.
it is relevant to the issues and is not excluded by the rules.
it is the means sanctioned by the rules of ascertaining the truth of a matter fact.
all of the above
(Answer: B; page 12; MPL 90)
A physician who testifies in court on matters he perceived in the course of a physician-patient relationship is a/an:
star witness
ordinary witness
expert witness
amicus curiae
(Answer: C; page 14; MPL 90)
Legal Medicine and Medical Jurisprudence

24 / 35
Included in the application of legal medicine in civil law are:
circumstances affecting criminal liability
crimes against chastity
paternity and filiation
all of the above
(Answer: C; page 1; MPL 30)
The following may qualify a crime from less grave to grave:
mitigating circumstances
aggravating circumstances
exempting circumstances
all of the above
(Answer: B; page 752; MPL 90)
One is exempted from criminal liabilities if he/she:
acts under the compulsion of an irresistible force.
acts under the impulse of an uncontrollable fear
fails to perform an act required by law when prevented by some lawful cause.
all of the above
(Answer: D; page 750; MPL 90)
Any person who in order to avoid evil or injury does an act which causes damage to another:
is exempt from criminal liability
does not incur criminal liability
guilty of the crime
considered negligent
(Answer: B; page 750; MPL 60)
Age becomes a mitigating circumstance in the following:
below 9 years old
below 18 years old
18 years old or over
50 years old or over
(Answer: B; page 752; MPL 60)
A person who is a habitual drunkard, committing a crime under the influence of alcohol will have the following under consideration in relation to the
crime committed:
alternative aggravating circumstance of intoxication
alternative mitigating circumstance of intoxication
exempting circumstance of intoxication
B and C only
(Answer: A; page 687; MPL 60)
A person who acts after sufficient provocation or threat on the part of the offended party, is acting under the following circumstance/s:
mitigating circumstance
exempting circumstance
justifying circumstance
aggravating circumstance
(Answer: A; page 752; MPL 33)
A person who kills another by poisoning should be charged with:
homicide
murder
parricide
genocide
(Answer: B; page 195; MPL 90)
Ground/s for dismissal of the charge or acquittal of the accused:
presumption of death
non-establishment of the identity of the offender
seven years' absence of a person
all of the above
(Answer: B; page 20; MPL 90)
In the identification of an individual the following may not easily be changed:
special preference for a certain form, texture or style
a clergyman by his robe
a swaying movement due to knock knee
color of hair
(Answer: C; page 44; MPL 90)
A person is identified through his mannerism in the manner of:
changing the eye
changing the hair
expression of the mouth while articulating
Legal Medicine and Medical Jurisprudence

25 / 35
occupational marks
(Answer: C; page 45; MPL 90)
A point of identification applicable to both living and dead before onset of decomposition:
callosities of the palm of the hand
age determination during infancy
height of a mature person
all of the above
(Answer: A; page 50; MPL 60)
Lack of free-flowing movement is characteristic of:
elderly person's handwriting
disguised writing
illiterate handwriting
writing of a person in doubt
(Answer: B; page 73; MPL 90)
Often, too much slanted loop letters is a characteristic of:
elderly person's handwriting
disguised writing
illiterate handwriting
writing of a person in doubt
(Answer: C; page 72; MPL 90)
Numerous change of direction and omission of parts of letters of stroke is a characteristic of:
elderly person's handwriting
disguised writing
illiterate handwriting
writing of a person in doubt
(Answer: A; page 72; MPL 90)
Under the Rules of Court, for an ordinary witness to qualify and express his opinion on an individual's handwriting he should:
have an intimate acquaintance respecting the mental sanity of a person
have seen the person write
be shown that he has some familiarity with the handwriting of the person in a way recognized by law.
all of the above
(Answer: C; page ; MPL 90)
For sex determination, the presence of Barr bodies is indicated by this test:
social test
gonadal test
chromosomal test
genital test
(Answer: C; page 88 ; MPL 60)
Crime wherein only a woman can be the victim:
adultery
abduction
concubinage
prostitution
(Answer: B; page 87 ; MPL 60)
Age as an exempting circumstance refers to a:
person under nine years of age
woman under twelve years of age
person over nine years of age and under fifteen
person less than 18 years of age.
(Answer: A; page 90; MPL 60)
Qualified seduction is the seduction of a:
woman who is single or a widow of good reputation, over twelve but under eighteen
virgin over twelve years and under eighteen years of age
woman under twelve years of age
virgin who is deprived of reason
(Answer: B; page 515; MPL 90)
It is always considered rape if carnal knowledge with a woman is done:
when the woman is deprived of reason
done when the woman is unconscious
done when the woman is under twelve years of age
all of the above
(Answer: D; page 501; MPL 90)
Luminescence test of blood is a form of:
chemical examination
Legal Medicine and Medical Jurisprudence

26 / 35
microscopic examination
physical examination
histologic examination
(Answer: C; page 97; MPL 60)
That which extinguishes civil personality is:
insanity
insolvency
death
terminal illness
(Answer: C; page 111; MPL 90)
The fall of temperature of 15 to 20 degrees Fahrenheit is considered as a sign of:
extreme anxiety
septic shock
death
rheumatic fever
(Answer: C; page 121; MPL 90)
When the body is in the water, the part first affected by putrefaction is the:
shoulders
abdomen
face
upper and lower extremities
(Answer: C; page 145 ; MPL 90)
The time within which rigor mortis sets in after death is:
1 - 2 weeks
24 - 48 hours
3 - 6 hours
2 - 3 hours
(Answer: C; page 127; MPL 60)
Under the Rules of Court, it is a disputable presumption that a person is dead if he/she is not heard from for:
two years
five years
seven years
ten years
(Answer: C; page 154; MPL 60)
The absentee shall not be presumed dead for the purpose of opening his succession till after an absence of:
two years
fiver years
seven years
ten years
(Answer: C; page 154 ; MPL 60)
The following officials of government are authorized to make death investigations:
clerk of court
city mayor
provincial governor
city fiscal
(Answer: D; page 156; MPL 90)
Medical legal investigations include the following:
investigation of the body of the victim
having access to all public records
administering oaths upon cases under investigation
all of the above
(Answer: A; page 164; MPL 90)
The strip method of conducting a search is done by:
searchers gathering at center and proceeding outwards along radii or spokes
searchers traversing first parallel to the base and then parallel to the south
blocking out the area in the form of a rectangle
searchers follow each other in the path in a spiral manner
(Answer: C; page 162; MPL 33)
This refers to an external examination of a dead body without incision:
non-official autopsy
official autopsy
post mortem examination
forensic examination
(Answer: C; page 163; MPL 60)
Legal Medicine and Medical Jurisprudence

27 / 35
Persons who are authorized to perform dissections include:
city fiscal
RTC judge
MTC judge
municipal health officers
(Answer: D; page 165; MPL 90)
Cause/s of death operating from the lung or pulmonary circulation is:
pressure on the respiratory tract due to tumor
fixation of the respiratory muscles from overstimulation of the spinal cord
pleurisy with effusion
acute bronchitis
(Answer: C; page 200; MPL 25)
Causes of death from asphyxia:
emphysema
pulmonary embolism
compression of the larynx
all of the above
(Answer: D; page 200; MPL 33)
The following are causes of coma:
increasing dyspnea
delirium
accumulation of mucous in the respiratory passages
none of the above
(Answer: D; page 200; MPL 90)
The following is/are symptom/s of coma:
accumulation of mucous in the respiratory passage
expiratory convulsion
persons becomes bluish and consciousness is lost
cold, clammy perspiration
(Answer: D; page 202; MPL 90)
Symptoms of asphyxia include:
lungs engorged with dark blood
relaxation of all sphincter muscles
accumulation of mucous in the respiratory passages
all of the above
(Answer: D; page 200; MPL 90)
Death due to pathological lesions is:
suicidal
homicidal
from syncope
all of the above
(Answer: C; page 200; MPL 90)
Causes of death from syncope include:
pulmonary embolism
marked expiratory effort
exhaustive diseases
uremia
(Answer: C; page 199; MPL 60)
A manner of death is:
suicidal
accidental
violent
medico-legal
(Answer: C; page 183; MPL 30)
Cause of death from cerebral apoplexy is:
occlusion of the coronary vessel
rupture of blood vessels inside the cranial cavity
inflammation of the covering membrane of the brain
laceration of the brain tissue
(Answer: B; page 189; MPL 90)
Symptoms of syncope include:
pupils are constricted
person becomes bluish
pulse at the wrist disappears
respiration is deep
Legal Medicine and Medical Jurisprudence

28 / 35
(Answer: C; page 199; MPL 90)
One of the distinguishing characteristics of the male pelvis is:
Sacrum is short and wide.
Greater sciatic notch is wide.
Obturator foramen is egg-shaped.
Pubic arch is wide and round.
(Answer: C; page 80; MPL 25)
A physician who testifies in court cannot be compelled to answer a question if the answer will:
not be relevant to the matter at issue.
antagonize the lawyer posing the question.
implicate a physician colleague, a relative or friend.
be self-incriminatory.
(Answer: D; page ; MPL 33)
Which of the following findings most strongly point to drowning as cause of death?
cyanosis
signs of physical struggle
marked congestion of trachea and bronchi
washerwoman's hands
(Answer: D; page 445; MPL 60)
Instantaneous rigidity of muscles which occurs at the moment of death:
heat stiffening
cold stiffening
cadaveric spasm
rigor mortis
(Answer: C; page 129; MPL 60)
Soon after death, blood begins to accumulate in the most dependent portions of the body giving rise to:
petecchiae
hematoma
hypostatic lividity
diffusion lividity
(Answer: C; page 132; MPL 90)
The feigning or simulation of a disease or injury is known as:
hysterical illness
hypochondriac tendency
malingering
delusion
(Answer: C; page 649; MPL 33)

One way to determine whether death has occurred is to apply ligature around the base of a finger and to observe whether there is change in the
color of the finger. This is known as:
Winslow's test
Icard's test
Magnus test
Diaphanous test
(Answer: C; page 118; MPL 33)
What distinguishes the entrance would from the exit wound caused by a gunshot?
The entrance wound is smaller than the exit wound.
The edges are everted in the entrance wound, and inverted in the exit wound.
The entrance wound has no definite shape while the exit wound is usually circular.
There is contusion collar in the exit wound and none in the entrance wound.
(Answer: A; page 354; MPL 90)
What is the legal responsibility of a physician when in the course of his practice he comes across a case of abused child?
show compassion and affection for the child
reprimand the parents
refer the parents for psychiatric counseling
report the case for the nearest unit of the Dept. of Social Service and Development.
(Answer: B; page 481; MPL 90)
Under the law an unembalmed body should be buried ___ hours after death:
12
24
48
72
(Answer: C; page 220; MPL 60)

Legal Medicine and Medical Jurisprudence

29 / 35
A physician should never examine or treat a hospitalized patient of another physician without the latter's knowledge and consent, EXCEPT when
the:
patient expressly asks him to do so in the absence of the attending physician.
physician is a close relative of the patient.
physician does not charge the patient professional fee.
attending physician is not readily available, and there is an emergency.
(Answer: D; page* ; MPL 90)
Vicarious liability of a physician refers to his responsibility for injury to his patient because of::
failure of the patient to follow instructions.
defective hospital equipment.
delay in attending to a patient.
negligent act committed by his nurse.
(Answer: D; page 233*; MPL 90)
Mrs. Z signs a legal document assigning her nephew to specifically decide whether a certain form of treatment is to be administered in case she
becomes incompetent. The document is known as:
living will
last will and testament
durable power of attorney
informed consent form
(Answer: C; page* ; MPL 33)
Marijuana is a psychotropic drug whose active ingredient is:
lysergic acid diethylamide
tetrahydrocannabinol
mescaline and psilocybin
myristic acid and phencyclidine
(Answer: B; page 667*; MPL 90)
Most medical practitioners seek membership in the Philippine Medical Association (PMA) because:
PMA membership is required in order to obtain a license to practice medicine.
PMA penalizes medical practitioners who are not members.
Practice in hospital setting requires PMA membership.
The Medicare Act provides that only PMA members may receive benefits under this law.
(Answer: D; page *; MPL 60)
The law that prohibits a physician from selling drug samples that he received free:
Medical Act
Pharmacy Act
Dangerous Drugs Act
Generics Act
(Answer: B; page 209*; MPL 60)
A prescription containing the phrase "NO SUBSTITUTION ALLOWED" is considered under the Generics Act as:
mandatory prescription
restrictive prescription
violative prescription
impossible prescription
(Answer: C; page* ; MPL 33)
Which of the following is classified as prohibited drug?
cocaine
amphetamine
seconal
atropine
(Answer: A; page 206*; MPL 90)
One instance when a physician can apply a procedure even if it is against the wishes of a patient is when the procedure is:
required by law
known to be safe
necessary for the good the patient.
routinely done on all patients with the same illness
(Answer: A; page 124*; MPL 33)
A patient suffering from painful incurable disease asked his physician to give him lethal dose of medication. Out of pity, the physician gave the
poison which subsequently led to the death of the patient. The physician is guilty of:
murder
homicide
giving assistance to suicide
dishonorable conduct
(Answer: D; page 209*; MPL 90)
Which is a ground for reprimand of a physician by the Board of Medicine?
Announcing his return from abroad and resumption of his medical practice in the Philippines.
Legal Medicine and Medical Jurisprudence

30 / 35
High mortality rate among his patients.
Serving as an expert medical witness in a court suit involving another physician.
Maligning a physician-colleague in front of other physicians.
(Answer: D; page* ; MPL 33)
A physician refers a patient to a specialist and received a share in the fee paid by the patient to the specialist. This is known as:
simple contractual fee
referral fee
contingent fee
fee splitting
(Answer: D; page 102*; MPL 90)
When an accepted method of medical treatment involves a hazard which may produce injury despite care exercised by the physician, the physician
may not be held liable should an injury occur under the doctrine:
doctrine of common fault
res ipsa loquitor
calculated risk
doctrine of foreseeability
(Answer: C; page 245*; MPL 60)
Any physician who in connection with the practice of medicine issued a false medical certificate is subject to penalty such as:
prision correctional in its medium periods
prision mayor
reclusion temporal
fine not exceeding 1,000 pesos
(Answer: D; page 200*; MPL 60)
Faith healing maybe considered a legitimate modality of treatment because it is:
allowed under the Medical Act.
covered under the constitutional guarantee of religious freedom.
perceived to be effective by many people
part of the culture of the people
(Answer: B; page 142*; MPL 33)
A hospital maybe held liable for the negligent act of a hospital resident physician under the doctrine of:
command responsibility
vicarious liability
res ipsa loquitor
assumption of risk
(Answer: B; page 233*; MPL 60)
Autonomy may not be exercised by a patient when:
there is extreme danger to the patient's health.
the patient' choice potentially harm innocent others.
the patient participates in a clinical trial.
a competent authority declares the patient to be incompetent.
(Answer: B; page 124*; MPL 33)
A certain procedure is allowed only under the Rule of Double Effect if the:
procedure results in both good and bad effects.
good effect were the direct causal result of the bad effect.
physician intends the bad effect in order to produce the good effect.
the good effect outweighs the bad effect.
(Answer: D; page ; MPL 60)
A physician may perform a diagnostic or therapeutic procedure without the consent of the patient if:
the physician believes the procedure is necessary.
the patient is merely being stubborn.
the patient is unconscious.
there is an emergency and there is no time to discuss the situation in order to seek consent.
(Answer: D; page 123*; MPL 33)
The privileged communication between patient and the physician must be guarded to protect the patient's right to:
autonomy
privacy
religious belief
refuse treatment
(Answer: B; page 149*; MPL 90)
For question numbers 86 - 89, match the lettered option indicating a sign of death with its appropriate description.
Algor mortis
Cadaveric spasm
Rigor mortis
Caloricity
Legal Medicine and Medical Jurisprudence

31 / 35
86. Post-mortem occurrence where the whole body becomes rigid due to contraction of the muscles.
(Answer: C; page 126; MPL 60)
87. Instantaneous rigidity of the muscles which occurs at the moment of death due to nervous tension.
(Answer: B; page 128; MPL 60)
88. Post-mortem rise in temperature of the body due to rapid and early putrefactive change
(Answer: D; page 122; MPL 90)
89. Post-mortem cooling of the body.
(Answer: A; page 122; MPL 90)
For question numbers 90 - 93, match the test with its appropriate description.
Phenolphthalein Test
Wagenhaar Test
Winslow Test
Diaphanous Test
90. Fingers are spread wide apart and finger webs are viewed through strong light.
(Answer: D; page 119; MPL 25)
91. Micro-chemical test for the presence of blood.
(Answer: B; page 100; MPL 25)
92. Method of detecting cessation of respiratory activity.
(Answer: C; page 120; MPL 25)
93. Chemical test for the presence of blood, also known as Kastle-Meyer Test
(Answer: A; page 100; MPL 25)
For question numbers 94 - 97, match the types of death with their appropriate description.
Cellular death
Brain death
Apparent death
Somatic death
94. May occur several hours after cessation of cardio-respiratory functions
(Answer: A; page 116; MPL 90)
D95. Occurs at the time of complete, continuous and persistent cessation of vital functions of the brain, heart and lungs.
(Answer: D; page 116; MPL 60)
C96. Transient cessation of vital functions
(Answer: C; page 116; MPL 90)
B97. Characterized by deep and irreversible coma, without the possibility of resuscitation
(Answer: B; page 116; MPL 90)
For question numbers 98 - 100, match the legal term with its appropriate definition.
Common law
Civil law
Criminal law
Jurisprudence
B98. Precepts that determine and regulate the relation between members of the family
(Answer: B; page 4; MPL 33)
A99. Composed of unwritten laws based on customs from time immemorial
(Answer: A; page 3; MPL 60)
D100. Practical science which investigates the nature and functions of law.
(Answer: D; page 4; MPL 90)

OUR LADY OF FATIMA UNIVERSITY
1. Patient was aware that physician was drunk yet he allowed the doctor to inject him:
A. doctrine of continuing negligence
C. res ipsa loquitor
B. vicarious liability
D. captain of the ship
2. Patient Mr. Lee promised to pay Dr. B one million pesos if he us cured of his cancer. The professional fee to be collected by B refers to:
A. contingent fee
B. package deal fee
C. retainers fee D. simple contractual fee
Legal Medicine and Medical Jurisprudence

32 / 35
3. HIV patient takes experimental drugs despite unknown side effects:
A. doctrine of informed consent
C. assumption of risk
B. doctrine of foreseeability
D. doctrine of contributory negligence
4. There is direct physical connection between the wrongful act of the Physician and the injury suffered by the patient:
A. Doctrine of proximate cause
C. “Sine Qua now” test
B. “But for” test
D. “Substantial factor” test
5. Principle applied when the medical fee is not specified:
A. “Asumpsit on quanthum merit” C. “Dura Lex sed Lex”
B. “Lex Loci contract”
D. “Sine Qua now”
6. Article 365 which deals with imprudence and negligence is a provision of:
A. Civil code
B. Medical Act 1959 C. Revised penal code D. Criminal Act of 1970
7. To which forum can an administrative complaint be filed against doctors?
A. Regional Trial Court
C. Department of health
B. National Bureau of Investigation
D. Professional Regulation Comission
8. Reprimand is a sanction imposed against a doctor when he is guilty of what liability:
A. Administrative liability B. Civil liability C. criminal liability D. no liability
9. If the license of a physician is suspended indefinitely, after how long can he apply for reinstatement?
A. 3 years B. 6 months C. 2 years D. it will depend on the pleasure of the Board of medicine
10. What is the degree of proof/evidence needed to convict physician of a crime?
A. Guilt beyond reasonable doubt C. Prosecution must prove that Dra. Gajardo got pregnant
B. Preponderance of evidence D. Prosecution must prove that Dr. Gimenez is a medical doctor
11. The study of licensure laws, regulatory laws and physician-patient relationship, is known in medical school curriculum as:
A. Legal MedicineB. Legal Ethics C. Medical Jurisprudence D. M<edical Ethics
12. Which of the following does not affect a termination of physician-patient relationship?
A. withdrawal of physician without patient consent
C. death of patient
B. recovery of patient
D. death of physician
13. A physician should never examine or treat hospitalized patient of another physician without the latter’s knowledge and consent except when:
A. the patient expressly asks him to do so in the absence of attending physician
B. he is a relative of the patient
C. he will not changed the patient a professional fee
D. in case of emergency, the attending physician is not readily available
14. A physician accused of criminal abortion was acquitted because:
A. he was a duly licensed medical practitioner
B. the woman plaintiff was emotionally unstable
C. the product of conception could not be produced in court
D. a pregnancy test done 3 days before the alleged abortion definitely ruled out pregnancy
CASE: Mr. A brought his 3 yrs old son to a hospital. Dr. B, the admitting physician admitted the pediatric case to the medicine ward because the
pediatric ward is full. During the night, the patient child was seen in the floor with a fractured skull resulting from a fall. It was found out that the bed was
without railings. Mr. A wanted to sue Dr. B and the hospital.
15. The doctrine applicable in this case would be:
A. Res ipsa loquitor
B. Osiensible agent
C. Forseeability D. Common knowledge
16. Suppose before admission Mr. A was informed of the absence of railings to protect the child from fall:
A. assumption of risk
C. res ipsa loquitor
B. continuing negligence
D. common knowledge
17. Suppose Mr. A has already previously and repeatedly complained of the absence of railings to Dr. B but no attention was given to the complaint,
which doctrine could be applicable?
A. assumption of risk
C. contributory negligence
B. continuing negligence
D. common knowledge
18. Suppose one day after an intramuscular injection a part of the needle was discovered protruding from the buttocks of the patient, what is the doctrine
applicable?
A. assumption of risk B. res ipsa loquitor C. contributory negligence D. continuing negligence
19. Originally there were railings on the bed of the patient. Mr A, the father removed the railings so that he himself can sleep on the bed. His child fell
with a broken skull. What is the doctrine applicable: A. res ipsa loquitor
C. contributory negligence
B. borrowed servant doctrine
D. common knowledge
20. Patient leaves the hospital against advice:
A. Continuing Negligence
C. Vicarious Liability
B. Res Iplsa Loquitor
D. Contributory Negligence
21. Common defense of the hospital are regards consultants liability to his private patients:
A. Independent Contractor
C. Captain of the ship doctrine
B. Joint and several liability
D. Vicarious liability
22. The reporting of treatment of serious physical injuries by a physician is:
A. Voluntary B. Mandatory
C. Case to case D. Every end of the month
23. Under the doctrine the hospital owner may be liable for the mistakes of the residents:
A. Borrowed servant doctrine
C. Vicarious liability
B. Captain of the ship
D. Fellow Servant doctrine
24. Which of the following laws is enforced by the state:
A. Substantive law
B. Moral law
C. Divine Law D. Natural Law
25. A terminally ill patient is given an overdose of sedatives without his knowledge. What crime if any is committed by the doctor:
A. Murder
C. Homicide
B. No crime because ethically allowed D. Assisted Suicide
26. The bone which is most informative regarding the sex of the owner of the skeleton being examined is the: A. Skull
B. Sternum
C. Pelvis
D. Femur
27. The willful deliberate and painless acceleration of death of a person:
A. Parricide
B. Euthanasia
C. Homicide
D. Infanticide
Legal Medicine and Medical Jurisprudence

33 / 35
28. The cause of death resulting from lack of skill, lack of foresight, and reckless imprudence is called:
A. Homicide
B. Accidental
C.
Murder
D. Negligence
29. When a body is the subject matter of a criminal investigation, it may be exhumed:
A. Anytime
B. 3 years after burial
C. 5 years after burial D. 10 years after burial
30. Any change for the position of the cadaver will not change the location of this type of cadaveric lividity: A. Hypostatic
B. Osmotic
C.
Diffusion
D. Alteration
31. Exhumation is also known as:
A. interment
B. ground burial
C. embalming
D. disinterment
32. Deep burial sites will make decomposition :
A. faster
B. slower
C. stationary
D. negligible
33. When blood accumulates in the most dependent portions of the dead body, it is called
A. primary6 flaccidity
C. post-mortem lividity
B. secondary flaccidity
D. algor mortis
34. A missing person is presumed dead if he is not heard from for:
A. 1 year
B. 2 years
C. 3 years
D. 4 years
35. Musketry, electrocution, and hanging are examples of
A. dead body disposal
C. sudden death
B. judicial death
D. asphyxial death
36. If the physician performs euthanasia without the patients knowledge, he is guilty of:
A. murder
C. merciful killing
B. nothing
D. assistance to the commission of suicide
37. If a 3-day old unidentified child was intentionally killed with cruelty, and during a calamity, the offender may be changed with:
A. infanticide
B. murder
C. homicide
D. parricide
38. Cadaveric rigidity is also known as
A. Rigor mortis
C. cadaveric spasm
B. Algor mortis
D. heat stiffening
39. The convict is put to death by a volley of fire from a firing squad in:
A. mugging
B. Muskety
C. garroting
D. murder
40. An unembalmed body should be buried not later than: (if the person died as a result of a natural cause):
A. 12 hrs B. 20 hrs.
C. 24 hrs.
D. 48 hrs.
41. An unembalmed body should be buried not later than: (if the person died as a result of a communicable disease): A. 12 hrs. B. 20 hrs. C. 24 hrs.
D. 48 hrs.
42. If one performed lawful act, and he did it with due care, and death occurred without intending to cause it, the death is
A. negligent
B. homicide
C. accidental
D. murder
43. Instantenous rigor is also known as
A. Rigor mortis
C. post-mortem muscular irritability
B. cadaveric spasm
D. post-mortem rigidity
44. It is the prominence of the superficial veins with reddish discoloration due to the process of decomposition which develops on both flanks of the
abdomen.
A. putrefaction
C. Adipose formation
B. Marbolization
D. Maceration
45. The microorganisms that plays a dominant role in decomposition is
A. bacillus mesenterious
C. chlostridium welchi
B. bacillus coli
D. bacillus proteus vulgaris
46. The following manner of death should be autopsied, EXCEPT
A. suicides
C. accidental death
B. death unattended by physician
D. death occurring in natural manner
47. A common law-husband who kills his common law wife with whom he has a 2 days old child can be prosecuted for
A. infanticide
C. homicide
B. parricide
D. impossible crime
48. Presidential Decree 169 mandates all doctors to report all cases involving:
A. slight and less physical injury
C. less serious and serious physical
B. mutilation
D. child abuse cases
49. When the color of the contusion changes to green, the contusion’s estimated age is
A. 2-4 days
B. 4-5 days
C. 7-10 days
D. 14-15 days
50. Fracture produced by a tangential or glancing approach of bullet:
A. pond fracture
C. gutter fracture
B. indented fracture
D. bursting fracture
51. This is a firearm which has a cylindrical magazine situated at the rear of the barrel which can accommodate five or six cartridge:
A. automatic pistol
C. rifle
B. revolver
D. shotgun
52. Its main function is the transformation of mechanical energy by the hit of the firing pin on the percussion cap to chemical energy:
A. primer
B. powder
C. bullet
D. projectile
53. If the diameter of the gunshot wound point of entry is 1 cm x 1 cm the probable caliber of the firearm used is
A. 0.38
B. 0.22
C. 0.45
D. 9 mm
54. The primer is usually made up of
A. lead, barium, antimony
C. lead and antimony
B. lead and barium
D. lead, barium and copper
55. Tattooing is also known as
A. stippling
B. soot
C. smudging
D. fouling
56. Factors which make the wound of entrance bigger than the caliber, except
A. deformity of the bullet which entered
B. bullet enters the skin sidewise
C. acute angular approach of the bullet
D. contraction of the elastic tissues of the skin
Legal Medicine and Medical Jurisprudence

34 / 35
57. “Pugilistic” position of a boxer is associated with
A. burns
B. drowning
C. asphyxia
D. gunshot wounds
58. The average time required for death in drowning is
A. 1-3 minutes
C. 5-10 minutes
B. 2-5 minutes
D. 3-6 minutes
59. This is quantitative determination of the chloride content of the blood in the right and left ventricle of the heart:
A. Gettler’s test
C. Winslow’s test
B. Takayama test
D. Loord’s test
60. The test which determines the whether semen is of human origin or not:
A. biological test of farnum
C. Berberio’s
B. Ganguli’s method
D. puramen reaction
61. It is an expressed acknowledgement by the accused in a criminal case of the truth of his guilt as the crime charged.
A. confession
D. all of the above
B. admission
E. none of the above
C. interrogation
62. The presence of weapon which is highly grasped by the hand of a victim of a shooting incident is an example of
A. Rigor mortis
C. cadaveric spasm
B. death stiffening
D. cadaveric rigidity
63. Cases when trauma or disease kill quickly that there is no opportunity for sequel or complication to develop is known as
A. proximate cause of death
C. immediate cause of death
B. secondary cause of death
D. complete cause of death
64. The most practical, useful and reliable means of preserving evidence is by
A. photography
C. description
B. sketching
D. manikin method
65. The comprehensive study of a dead body performed by trained physician employing recognized procedures and techniques is known as
A. Post-mortem examination
C. medical examination
B. autopsy
D. Post-mortem dissection
66. The principle used in the Magnus test for determining somatic death is
A. no change in color of the finger
B. no change in the temperature of the finger
C. change in the color of the finger
D. change in the temperature of the torniqueted finger
67. When the body surface of corpse is pressed it leaves a flattened area due to
A. absence of edema
C. precipitated skin proteins
B. loss of elasticity of the skin
D. absence of blood
68. Cutis Galina or Washer woman’s hands and feet on the corpse signifies that:
A. the body has been in water for sometime
B. the body is that of a laundry woman
C. the body died for drowning
D. the body died for syphilis
69. Permission to disinter remains of persons who died of dangerous communicable diseases maybe granted after a burial period of
A. 4 years
B. 5 years
C. 6 years
D. 7 years
70. This is a medical evidence made known or addressed to the senses of the court no limited to the sense of vision, but is intended to the sense of
hearing, taste, smell and touch
A. experimental evidence
C. autoptic or real evidence
B. documentary evidence
D. corpus evidence
71. The most prominent sign of death is
A progressive fall of the body temperature
B. cessation of heart action and circulation
C. cessation of respiration
D. insensibility of body loss of power to move
72. The study of licensure laws, regulatory laws and physician-patient relationship, is known in medical school curriculum as:
A. Legal Medicine
C. Medical Jurisprudence
B. Legal Ethics
D. Medical Ethics
73. A mental process whereby one or more persons appraise a situation and make a decision based on their judgement that may or may not lead to
action is called:
A. Treatment
B. Diagnosis
C. Operation
D. Analysis
74. Faith healing is allowed as an norm of treatment modality in the Philippines. This is allowed on the basis of Constitutional guarantee on:
A. freedom of origin
C. freedom of the press
B. freedom of expression
D. freedom of sexual expression
75. The nature of liability against Dr. a would be:
A. administrative
B. civil
C. criminal
D. ethical
76. The venue with which to file the case in case Mr. Ty demands P1 million pesos as payment would be:
A. Regional Trial Court
C. Department of health
B. Professional Regulation commission
D. All of the above
77. The cost of the procedure to re-attach of Mr. Ty that was incided would be in nature of:
A. moral damages
C. exemplary damages
B. actual damages
D. all of the above
78. Mr. Ty suffered sleepless nights and wounded feelings as well as besmirched reputation.
This kind of damage is referred to as:
A. actual damage
C. exemplary damage
B. moral damage
D. attorney’s fees
79. Dr. A uttered vexing words against Dr. B defaming his personality and dignity. Dr. A maybe guilty of:
A. Libel B. Slander
C. rumor mongering
D. any of the above
Legal Medicine and Medical Jurisprudence

35 / 35
80. When a surgeon operates on a patient without consent, this constitutes an assault and even without proof of injury resulting, the plaintiff may recover
what type of damage?
A. Nominal
B. Moderate
C. Exemplary
D. Liquidated
81. When is patient’s consent necessary?
A. in an emergency, patient need tracheostomy
B. in a women diagnosed with ectopic pregnancy, it turns out be a case of acute appendicitis and the surgeon decides to remove the
appendix
C. During an appendectomy, the surgeon discovers and abnormal ovary and desiced to remove it.
D. The colon was perforated during simoidoscopy and the surgeon decides on laparotomy to undertake repair
82. A physician may perform a diagnostic or therapeutic procedure without the consent of the patient or his relatives.
A. when the physician believes the procedure is necessary
B. when the patient is merely being stubborn
C. when the patient is unconscious
D. in an acute emergency when there in no time to discuss the situation in order to seek consent
83. A physician may apply a procedure even it is against the wishes of the patient if
A. the procedure is known to be safe
B. the procedure is necessary for the good of the patient
C. the procedure is required by law
D. there is implied consent by he patient’s spouse
84. The following are provisions of Penal Law wherein a physician may be hold criminally liable. Which provision specifically mention the physician as
the wrong-doer?
A. defamation
B. criminal negligence and imprudence
C. violation of the Dangerous Drug Act
D. refusal to render treatment in emergency cases
85. The testimony of an expert witness is NOT needed to prove a negligent act when which doctrine is applicable? A. res ipsa loquitor
C. ostensible agent
B. borrowed servant
D. captain of the ship
86. Instances when a physician may NOT be held liable for abandonment.
A. failure to visit the patient sufficiently after believing that the patient needed no further treatment
B. failure to provide follow-up attention
C. refusal to attend a case for which he has assumed responsibility
D. failure to arrange for a substitute during the attending physician’s absence
87. A physician cannot be held liable for the ill effect of his management procedure if
A. he can show that he applied his knowledge and skill with diligence and care
B. the doctrine of res ipsa loquitor is applicable
C. the patient contributed to the ill-effects
D. the procedure is by its nature risky
88. A physician who gives an overdose of a drug may be charged for
A. Immorality
C. gross negligence
B. Dishonorable conduct
D. incompetence
89. One of the grounds for the suspension or revocation of the certificate of registration of a physician on account of a criminal act is
A. immoral or dishonorable conduct
B. violation of the Code of ethics
C. conviction by court of an offense involving moral turpitude
D. false, extravagant or unethical advertisement
90. A balikbayan physician puts up a sign board at his clinic. Which is unethical?
A. “Disease of Women and Children” C. consultation hours 9-11 a.m.
B. “Trained at Mayo Clinic”
D. consultation by appointment only
91. The mistress of a married physician sought redress from the Board of Medicine when the physician left her for another woman. Which statement is
VALID?
A. she can charge the physician for immorality C. she has no right to complain
B. she can accuse him for adultery
D. she can charge him of abandonment
92. An unmarried physician was discovered having sex with an unmarried nurse. He may be charged before the Board of medicine for
A. Seduction
B. Immorality
C. Adultery
D. reckless imprudence
93. The prohibition and imposition of penalty on physicians who sell medical samples gratuitously given to them by drug promoters is embodied in
A. Medical Act 1959 as amended
C. Revised Penal Code
B. Revised Administrative Code
D. Pharmacy Law
94. The element of deceit is required in case of:
A. forcible obduction
C. simple seduction
B. consented obduction
D. qualified seduction
95. The spinning motion of the bullet, its rough surface, and the imagination of the skin, all produce the:
A. Tattooing
B. Rifling
C.
Contusion collar D. smudging
96. The skin is whitened, macerated and wrinkled in:
A. cutis anserina
C. washerwoman’s skin
B. saponification
D. contact flattening
97. Sexual intercourse is NOT necessary in this crime:
A. Abduction
B. Seduction
C. Adultery
D. Concubinage
98. Virginity is a requirement in this crime:
A. simple seduction
C. qualified seduction
B. forcible abduction
D. white slave trade
99. The laceration involving more than half of the hymenal height but does not reach the hymenal based is called:
A. Superficial B. Deep
C. Complete
D. Complicated
100. Stippling is caused by:
A. Flame
B. gunpowder residue C. smoke
D. bullet
Legal Medicine and Medical Jurisprudence

Sponsor Documents

Or use your account on DocShare.tips

Hide

Forgot your password?

Or register your new account on DocShare.tips

Hide

Lost your password? Please enter your email address. You will receive a link to create a new password.

Back to log-in

Close